Find a set of size $m$ consisting of vectors from $mathbbR^n$, so that each of its subsets of size $n$ is linearly independent.

The name of the pictureThe name of the pictureThe name of the pictureClash Royale CLAN TAG#URR8PPP











up vote
0
down vote

favorite












Find a set of size $m$ consisting of vectors from $mathbbR^n$, so that each of its subsets of size $n$ is linearly independent.



Also, what is the maximum $m$ for which this is possible?



So here is my attempt at calculating an upper-bound by induction: $binomn+12$. We can assume $n$ of the vectors are our standard basis. Let the rest of the set be $B$. Now from the standard basis take $varepsilon_1$, this vector and subset of $B$ of size $n-1$ must together form an independent set. Hence $B$ has size at most $binomn2$. Which proves our claim. But I am unable to construct an example with such many vectors or attain a lower upper-bound.



It was pointed out in the comments this proof is wrong, as it doesn't hold for n = 2.







share|cite|improve this question

















  • 5




    Consider $Bbb R^2$ and consider the set of vectors $A=left(x,1)~:~xinBbb Rright$. It should be clear that every pair of vectors in $A$ are linearly independent and it should also be clear that $A$ is uncountably infinite in size.
    – JMoravitz
    Jul 23 at 22:53










  • @JMoravitz Can you construct such an arbitrarily set for any n?
    – J. Doe
    Jul 23 at 23:06






  • 1




    Yes. Consider the set $A=(1,x,x^2,x^3,dots,x^n-1)~:~xinBbb Rsubseteq Bbb R^n$. Any set of $n$ distinct elements of $A$ will be linearly independent. See Vandermonde Matrix on wikipedia for additional details.
    – JMoravitz
    Jul 23 at 23:28















up vote
0
down vote

favorite












Find a set of size $m$ consisting of vectors from $mathbbR^n$, so that each of its subsets of size $n$ is linearly independent.



Also, what is the maximum $m$ for which this is possible?



So here is my attempt at calculating an upper-bound by induction: $binomn+12$. We can assume $n$ of the vectors are our standard basis. Let the rest of the set be $B$. Now from the standard basis take $varepsilon_1$, this vector and subset of $B$ of size $n-1$ must together form an independent set. Hence $B$ has size at most $binomn2$. Which proves our claim. But I am unable to construct an example with such many vectors or attain a lower upper-bound.



It was pointed out in the comments this proof is wrong, as it doesn't hold for n = 2.







share|cite|improve this question

















  • 5




    Consider $Bbb R^2$ and consider the set of vectors $A=left(x,1)~:~xinBbb Rright$. It should be clear that every pair of vectors in $A$ are linearly independent and it should also be clear that $A$ is uncountably infinite in size.
    – JMoravitz
    Jul 23 at 22:53










  • @JMoravitz Can you construct such an arbitrarily set for any n?
    – J. Doe
    Jul 23 at 23:06






  • 1




    Yes. Consider the set $A=(1,x,x^2,x^3,dots,x^n-1)~:~xinBbb Rsubseteq Bbb R^n$. Any set of $n$ distinct elements of $A$ will be linearly independent. See Vandermonde Matrix on wikipedia for additional details.
    – JMoravitz
    Jul 23 at 23:28













up vote
0
down vote

favorite









up vote
0
down vote

favorite











Find a set of size $m$ consisting of vectors from $mathbbR^n$, so that each of its subsets of size $n$ is linearly independent.



Also, what is the maximum $m$ for which this is possible?



So here is my attempt at calculating an upper-bound by induction: $binomn+12$. We can assume $n$ of the vectors are our standard basis. Let the rest of the set be $B$. Now from the standard basis take $varepsilon_1$, this vector and subset of $B$ of size $n-1$ must together form an independent set. Hence $B$ has size at most $binomn2$. Which proves our claim. But I am unable to construct an example with such many vectors or attain a lower upper-bound.



It was pointed out in the comments this proof is wrong, as it doesn't hold for n = 2.







share|cite|improve this question













Find a set of size $m$ consisting of vectors from $mathbbR^n$, so that each of its subsets of size $n$ is linearly independent.



Also, what is the maximum $m$ for which this is possible?



So here is my attempt at calculating an upper-bound by induction: $binomn+12$. We can assume $n$ of the vectors are our standard basis. Let the rest of the set be $B$. Now from the standard basis take $varepsilon_1$, this vector and subset of $B$ of size $n-1$ must together form an independent set. Hence $B$ has size at most $binomn2$. Which proves our claim. But I am unable to construct an example with such many vectors or attain a lower upper-bound.



It was pointed out in the comments this proof is wrong, as it doesn't hold for n = 2.









share|cite|improve this question












share|cite|improve this question




share|cite|improve this question








edited Jul 23 at 23:05
























asked Jul 23 at 22:48









J. Doe

11




11







  • 5




    Consider $Bbb R^2$ and consider the set of vectors $A=left(x,1)~:~xinBbb Rright$. It should be clear that every pair of vectors in $A$ are linearly independent and it should also be clear that $A$ is uncountably infinite in size.
    – JMoravitz
    Jul 23 at 22:53










  • @JMoravitz Can you construct such an arbitrarily set for any n?
    – J. Doe
    Jul 23 at 23:06






  • 1




    Yes. Consider the set $A=(1,x,x^2,x^3,dots,x^n-1)~:~xinBbb Rsubseteq Bbb R^n$. Any set of $n$ distinct elements of $A$ will be linearly independent. See Vandermonde Matrix on wikipedia for additional details.
    – JMoravitz
    Jul 23 at 23:28













  • 5




    Consider $Bbb R^2$ and consider the set of vectors $A=left(x,1)~:~xinBbb Rright$. It should be clear that every pair of vectors in $A$ are linearly independent and it should also be clear that $A$ is uncountably infinite in size.
    – JMoravitz
    Jul 23 at 22:53










  • @JMoravitz Can you construct such an arbitrarily set for any n?
    – J. Doe
    Jul 23 at 23:06






  • 1




    Yes. Consider the set $A=(1,x,x^2,x^3,dots,x^n-1)~:~xinBbb Rsubseteq Bbb R^n$. Any set of $n$ distinct elements of $A$ will be linearly independent. See Vandermonde Matrix on wikipedia for additional details.
    – JMoravitz
    Jul 23 at 23:28








5




5




Consider $Bbb R^2$ and consider the set of vectors $A=left(x,1)~:~xinBbb Rright$. It should be clear that every pair of vectors in $A$ are linearly independent and it should also be clear that $A$ is uncountably infinite in size.
– JMoravitz
Jul 23 at 22:53




Consider $Bbb R^2$ and consider the set of vectors $A=left(x,1)~:~xinBbb Rright$. It should be clear that every pair of vectors in $A$ are linearly independent and it should also be clear that $A$ is uncountably infinite in size.
– JMoravitz
Jul 23 at 22:53












@JMoravitz Can you construct such an arbitrarily set for any n?
– J. Doe
Jul 23 at 23:06




@JMoravitz Can you construct such an arbitrarily set for any n?
– J. Doe
Jul 23 at 23:06




1




1




Yes. Consider the set $A=(1,x,x^2,x^3,dots,x^n-1)~:~xinBbb Rsubseteq Bbb R^n$. Any set of $n$ distinct elements of $A$ will be linearly independent. See Vandermonde Matrix on wikipedia for additional details.
– JMoravitz
Jul 23 at 23:28





Yes. Consider the set $A=(1,x,x^2,x^3,dots,x^n-1)~:~xinBbb Rsubseteq Bbb R^n$. Any set of $n$ distinct elements of $A$ will be linearly independent. See Vandermonde Matrix on wikipedia for additional details.
– JMoravitz
Jul 23 at 23:28











1 Answer
1






active

oldest

votes

















up vote
1
down vote













An important example is the so called "moment curve" of points $(1,t,t^2,dots t^n-1)$. If we choose all of our $m$ points to be on this curve, the claim is that any $n$ of them will be linearly independent.



So see this, suppose not, then there would be a linear functional $a_1x_1+a_2x_2+ dots a_nx_n = 0$ satisfied by all of them with out all of the $a_i$'s being $0$. But then the polynomial $a_1+a_2x+a_3x^2+ dots a_nx^n-1$ would be a degree $n-1$ polynomial with $n$ roots, a contradiction.



Of course if you randomly choose $m$ points say uniformly within a ball or from a gaussian distribution they will also have this property you want with probability $1$...






share|cite|improve this answer





















    Your Answer




    StackExchange.ifUsing("editor", function ()
    return StackExchange.using("mathjaxEditing", function ()
    StackExchange.MarkdownEditor.creationCallbacks.add(function (editor, postfix)
    StackExchange.mathjaxEditing.prepareWmdForMathJax(editor, postfix, [["$", "$"], ["\\(","\\)"]]);
    );
    );
    , "mathjax-editing");

    StackExchange.ready(function()
    var channelOptions =
    tags: "".split(" "),
    id: "69"
    ;
    initTagRenderer("".split(" "), "".split(" "), channelOptions);

    StackExchange.using("externalEditor", function()
    // Have to fire editor after snippets, if snippets enabled
    if (StackExchange.settings.snippets.snippetsEnabled)
    StackExchange.using("snippets", function()
    createEditor();
    );

    else
    createEditor();

    );

    function createEditor()
    StackExchange.prepareEditor(
    heartbeatType: 'answer',
    convertImagesToLinks: true,
    noModals: false,
    showLowRepImageUploadWarning: true,
    reputationToPostImages: 10,
    bindNavPrevention: true,
    postfix: "",
    noCode: true, onDemand: true,
    discardSelector: ".discard-answer"
    ,immediatelyShowMarkdownHelp:true
    );



    );








     

    draft saved


    draft discarded


















    StackExchange.ready(
    function ()
    StackExchange.openid.initPostLogin('.new-post-login', 'https%3a%2f%2fmath.stackexchange.com%2fquestions%2f2860842%2ffind-a-set-of-size-m-consisting-of-vectors-from-mathbbrn-so-that-each-o%23new-answer', 'question_page');

    );

    Post as a guest






























    1 Answer
    1






    active

    oldest

    votes








    1 Answer
    1






    active

    oldest

    votes









    active

    oldest

    votes






    active

    oldest

    votes








    up vote
    1
    down vote













    An important example is the so called "moment curve" of points $(1,t,t^2,dots t^n-1)$. If we choose all of our $m$ points to be on this curve, the claim is that any $n$ of them will be linearly independent.



    So see this, suppose not, then there would be a linear functional $a_1x_1+a_2x_2+ dots a_nx_n = 0$ satisfied by all of them with out all of the $a_i$'s being $0$. But then the polynomial $a_1+a_2x+a_3x^2+ dots a_nx^n-1$ would be a degree $n-1$ polynomial with $n$ roots, a contradiction.



    Of course if you randomly choose $m$ points say uniformly within a ball or from a gaussian distribution they will also have this property you want with probability $1$...






    share|cite|improve this answer

























      up vote
      1
      down vote













      An important example is the so called "moment curve" of points $(1,t,t^2,dots t^n-1)$. If we choose all of our $m$ points to be on this curve, the claim is that any $n$ of them will be linearly independent.



      So see this, suppose not, then there would be a linear functional $a_1x_1+a_2x_2+ dots a_nx_n = 0$ satisfied by all of them with out all of the $a_i$'s being $0$. But then the polynomial $a_1+a_2x+a_3x^2+ dots a_nx^n-1$ would be a degree $n-1$ polynomial with $n$ roots, a contradiction.



      Of course if you randomly choose $m$ points say uniformly within a ball or from a gaussian distribution they will also have this property you want with probability $1$...






      share|cite|improve this answer























        up vote
        1
        down vote










        up vote
        1
        down vote









        An important example is the so called "moment curve" of points $(1,t,t^2,dots t^n-1)$. If we choose all of our $m$ points to be on this curve, the claim is that any $n$ of them will be linearly independent.



        So see this, suppose not, then there would be a linear functional $a_1x_1+a_2x_2+ dots a_nx_n = 0$ satisfied by all of them with out all of the $a_i$'s being $0$. But then the polynomial $a_1+a_2x+a_3x^2+ dots a_nx^n-1$ would be a degree $n-1$ polynomial with $n$ roots, a contradiction.



        Of course if you randomly choose $m$ points say uniformly within a ball or from a gaussian distribution they will also have this property you want with probability $1$...






        share|cite|improve this answer













        An important example is the so called "moment curve" of points $(1,t,t^2,dots t^n-1)$. If we choose all of our $m$ points to be on this curve, the claim is that any $n$ of them will be linearly independent.



        So see this, suppose not, then there would be a linear functional $a_1x_1+a_2x_2+ dots a_nx_n = 0$ satisfied by all of them with out all of the $a_i$'s being $0$. But then the polynomial $a_1+a_2x+a_3x^2+ dots a_nx^n-1$ would be a degree $n-1$ polynomial with $n$ roots, a contradiction.



        Of course if you randomly choose $m$ points say uniformly within a ball or from a gaussian distribution they will also have this property you want with probability $1$...







        share|cite|improve this answer













        share|cite|improve this answer



        share|cite|improve this answer











        answered Jul 23 at 23:27









        Nate

        6,57011020




        6,57011020






















             

            draft saved


            draft discarded


























             


            draft saved


            draft discarded














            StackExchange.ready(
            function ()
            StackExchange.openid.initPostLogin('.new-post-login', 'https%3a%2f%2fmath.stackexchange.com%2fquestions%2f2860842%2ffind-a-set-of-size-m-consisting-of-vectors-from-mathbbrn-so-that-each-o%23new-answer', 'question_page');

            );

            Post as a guest













































































            Comments

            Popular posts from this blog

            Color the edges and diagonals of a regular polygon

            Relationship between determinant of matrix and determinant of adjoint?

            What is the equation of a 3D cone with generalised tilt?